Stay ahead of learning milestones! Enroll in a class over the summer!

G
Topic
First Poster
Last Poster
Coaxal Circles
fattypiggy123   29
N 4 minutes ago by sttsmet
Source: China TSTST Test 2 Day 1 Q3
Let $ABCD$ be a quadrilateral and let $l$ be a line. Let $l$ intersect the lines $AB,CD,BC,DA,AC,BD$ at points $X,X',Y,Y',Z,Z'$ respectively. Given that these six points on $l$ are in the order $X,Y,Z,X',Y',Z'$, show that the circles with diameter $XX',YY',ZZ'$ are coaxal.
29 replies
fattypiggy123
Mar 13, 2017
sttsmet
4 minutes ago
No more topics!
Rhombus EVAN
62861   71
N Apr 12, 2025 by ihategeo_1969
Source: USA January TST for IMO 2017, Problem 2
Let $ABC$ be a triangle with altitude $\overline{AE}$. The $A$-excircle touches $\overline{BC}$ at $D$, and intersects the circumcircle at two points $F$ and $G$. Prove that one can select points $V$ and $N$ on lines $DG$ and $DF$ such that quadrilateral $EVAN$ is a rhombus.

Danielle Wang and Evan Chen
71 replies
62861
Feb 23, 2017
ihategeo_1969
Apr 12, 2025
Rhombus EVAN
G H J
Source: USA January TST for IMO 2017, Problem 2
The post below has been deleted. Click to close.
This post has been deleted. Click here to see post.
IAmTheHazard
5001 posts
#64 • 1 Y
Y by cubres
stupid. i dislike it strongly

Let $M$ be the midpoint of $\overline{AE}$ and let $\overline{DM}$ intersect the $A$-excircle again at $T$. Let $I$ and $I_A$ denote the incenter and $A$-excenter, respectively, of $\triangle ABC$. It suffices to prove that $DFXG$ is harmonic, whence projecting through $D$ yields the desired result. Let $N$ be the midpoint of $\overline{DX}$ and let $T$ be the intersection of the tangents to the $A$-excircle at $D$ and $X$. It suffices to show that $T$ lies on $\overline{FG}$.

It is well-known (by homothety) that $I$ lies on $\overline{DM}$. Then, by incenter-excenter, $N$ lies on $(IBCI_A)$, which has diameter $\overline{II_A}$. Let the radical axis of $(IBCI_A)$ and the $A$-excircle be $\ell$, which is also the polar of $I$ with respect to the excircle. Since $I$ lies on the polar of $T$, $T$ lies on $\ell$. By radical center on $(ABC)$, $(IBCI_A)$, and the $A$-excircle, $\overline{BC}$, $\overline{FG}$, and $\ell$ concur, hence $T$ lies on $\overline{FG}$ as well; done. $\blacksquare$

Remark: During the solve process I went through every single ARCH hint (not at the same time) only to discover, on each of them, that I had already done what the hint told me to :thonk:
This post has been edited 2 times. Last edited by IAmTheHazard, Sep 25, 2023, 4:50 PM
Z K Y
The post below has been deleted. Click to close.
This post has been deleted. Click here to see post.
Shreyasharma
676 posts
#65 • 1 Y
Y by cubres
Let $M$ be the midpoint of $AE$, and let $\omega$ denote the $A$-excircle. Let $X = MD \cap \omega$, and define $V$ and $N$ as the intersections of $DF$ and $DG$ with the perpendicular bisector of $AE$. Let the tangents to $\omega$ from $D$ and $X$ meet at $T$. Projecting through $D$ it suffices to show, $$-1 = (VN,MP_\infty) \overset{D}{=} (FG,XD)$$or equivalently that $GXGD$ is harmonic. It then suffices to show $T-F-G$.

Before we begin we provide a well-known lemma: $MD$ passes through $I$, the incenter of $ABC$.

Now letting $L$ be the midpoint of $DX$, we find that $\angle KLM = 90$, where $K$ is the $A$-excenter. Then if $Y, Z = (IBC) \cap \omega$, we find by radical center that $BC$, $GF$ and $YZ$ concur. Then it would suffice to show that $T \in YZ$. However note by La Hire's it suffices to show that $I$ lies on the polar of $T$, namely $DX$, which is clearly true.
Z K Y
The post below has been deleted. Click to close.
This post has been deleted. Click here to see post.
Aaryabhatta1
1631 posts
#66 • 1 Y
Y by cubres
If $EVAN$ is a rhombus $EV = AV, EN = VN$ imply that $V, N$ lie on the perpendicular bisector of $AE.$ Define $V$ and $N$ to be the intersections with $DG, DF.$ It suffices to prove that $AV = AN$, equivalently the midpoint of $M$ of $AE$ is the midpoint of $AV, AN.$

Lemma 1: Let $K, L$ be the other extouch points of the $A$-excircle with $\overline{AB}, \overline{AC}.$ Define $D'= \overline{KL} \cap \overline{BC}.$ Then $(B, C; D, D') = -1.$
Proof

Let $U$ be the intersection of the $A$-angle bisector with $BC.$ Since $\angle IBI_A = 90^\circ$ and $\angle ABI = \angle UBI,$ we have $(A, U; I, I_A) = -1.$

Now consider the polars, w.r.t the $A$-excircle, of these four points. Since they are collinear, their polars are concurrent to the pole of $\overline{AI}$ by LaHire. We can see that the polars form an angle harmonic bundle. So consider their projections onto $BC$; $\text{polar}(A) = \overline{KL}$ means $A \to D'$, $D \in \text{polar}(U)$ so $U \to D$, and $I_A \to \infty.$ Since the projection is a harmonic bundle, $I$ must map to the harmonic conjugate of $\infty$, which is $R$ the midpoint of $DD'.$

So $R \in \text{polar}(I)$ and by LaHire, $I \in \text{polar}(R).$ Define $H$ to be the other tangency of $R$ to the excircle. Then $I \in \overline{DH}.$

Lemma 2: $M, I, D$ are collinear.
Proof

Lemma 3: Let $R$ be the midpoint of $DD'$, $R$ lies on $\overline{GF}.$
Proof

Now we can finish; $MDH$ are collinear, $RD, RH$ are tangents to the excircle, and $R, G, F$ are collinear. So,
$$(N, V; M, \infty) \stackrel{D}{=} (F, G; H, D) = -1 $$and $M$ is the midpoint of $N, V.$ $\blacksquare$
Attachments:
This post has been edited 4 times. Last edited by Aaryabhatta1, Dec 23, 2023, 5:54 AM
Z K Y
The post below has been deleted. Click to close.
This post has been deleted. Click here to see post.
ihatemath123
3442 posts
#67 • 3 Y
Y by OronSH, GrantStar, cubres
It's well known that there exists some homothety centered at $D$ which sends the midpoint of $\overline{AE}$ to the incenter $I$ of $\triangle ABC$ and sends $E$ to the incircle touchpoint with side $\overline{BC}$. Let $\ell$ be the line through $I$ parallel to $\overline{BC}$.

It suffices to show that if lines $FD$ and $GD$ intersect $\ell$ at $X$ and $Y$ respectively, the midpoint of $\overline{XY}$ is $I$.

Let $P$ and $Q$ be the intersections of lines $FD$ and $GD$ with $(ABC)$, respectively. By Reim's theorem, $BCQP$ is an isosceles trapezoid. In fact, we will show, via length calculations, that line $PQ$ is exactly halfway between parallel lines $BC$ and $XY$. In particular, the distance from $Q$ to line $\overline{BC}$ is
\begin{align*}
& \sin ( \angle QDC ) \cdot QD \\
= ~ & \sin (\angle DFG) \cdot \frac{DC \cdot BD}{DG} \\
= ~ & \frac{DC \cdot BD}{\frac{DG}{\sin (\angle DFG)}} \\
= ~ & \frac{DC \cdot BD}{2r_A} \\
= ~ & \frac{r}{2},
\end{align*}as desired. elaboration on the last step

Thus, a homothety of factor $2$ centered at $D$ sends $P$ to $X$ and $Q$ to $Y$; thus, the perpendicular bisector of $\overline{PQ}$ is sent to the perpendicular bisector of $\overline{XY}$. But the perpendicular bisector of $\overline{PQ}$ is the perpendicular bisector of $\overline{BC}$; a homothety of factor $2$ centered at $D$ sends the perpendicular bisector of $\overline{BC}$ to the line through $I$ perpendicular to $\ell$. In particular, $I$ is the midpoint of $\overline{XY}$ as desired.
Z K Y
The post below has been deleted. Click to close.
This post has been deleted. Click here to see post.
shendrew7
793 posts
#68 • 1 Y
Y by cubres
We set $N$ and $V$ to be the intersections of the perpendicular bisector of $AE$ with $DF$ and $DG$. It remains to show
\[-1 = (VN;L \infty) \overset{D}{=} (GF;KD),\]
where $L$ is the midpoint of $AE$ and $K = LD \cap \omega_A$. In other words, we need $XK$ to be tangent to the excircle, where $X = BC \cap GF$.

From Midpoint of the Altitude lemma, $I$ lies on $KL$. Suppose $(II_A)$ meets the excircle at two points $P$ and $Q$. Radical axis theorem on the excircle, the circumcircle, and $(II_A)$ tells us $PQ$ passes through $X$. Finally, noting that $IP$ and $IQ$ are tangents to the excircle, $KPDQ$ is harmonic, which implies the desired. $\blacksquare$
Z K Y
The post below has been deleted. Click to close.
This post has been deleted. Click here to see post.
fuzimiao2013
3301 posts
#69 • 1 Y
Y by cubres
Got the exact same solution as EVAN on accident :P

Let $M$ be the midpoint of $AE$. It is well-known that $M$, $I$, and $D$ are collinear. Let $MID$ intersect the $A$-excircle again at $P$ and let $(II_A)$ intersect the $A$-excircle at $X$ and $Y$. By radical axis, we know that $BC$, $XY$, $FG$ are concurrent. Call this concurrency point $K$. We claim that $N$ is the intersection of the perpendicular bisector of $AE$ with $DF$, and $V$ similarly with $DG$.

Take all poles and polars wrt the $A$-excircle. Since $KD$ is tangent to the $A$-excircle, $D$ lies on the polar of $K$. Also, note that $\angle IXI_A = IYI_A = 90^\circ$, hence $I$ is the pole of $XYK$, and by La Hire's, this implies that $I$ is on the polar of $K$. Hence the polar of $K$ is $MIDP$ and so
\[
    -1 = (DP;FG) \stackrel D= (P_\infty M;NV)
\]where $P_\infty$ is the point of infinity along line $BC$. Thus $M$ is the midpoint of $AE$, $NV$, and $AE \perp NV$, and so $EVAN$ is a rhombus.
Z K Y
The post below has been deleted. Click to close.
This post has been deleted. Click here to see post.
Scilyse
387 posts
#70 • 1 Y
Y by cubres
[asy]
    import olympiad; import cse5;
    defaultpen(fontsize(10pt));
    usepackage("amsmath"); usepackage("amssymb"); usepackage("gensymb"); usepackage("textcomp");
    size(8cm);

    pair MRA (pair B, pair A, pair C, real r, pen q=anglepen) {
        r=r/2;
        pair Bp=unit(B-A)*r+A;
        pair Cp=unit(C-A)*r+A;
        pair P=Bp+Cp-A;
        D(Bp--P--Cp,q);
        return A;
    }
    
    pointpen=black+linewidth(2);
    pen polyline=linewidth(pathpen)+rgb(0.6,0.2,0);
    pen polyfill=polyline+opacity(0.1);
    pen angleline=linewidth(pathpen)+rgb(0,0.4,0);
    pen anglefill=angleline+opacity(0.4);
    markscalefactor=0.01;
    size(15cm);
    
    pair A=dir(120),B=dir(210),C=dir(330);
    // filldraw(A--B--C--cycle,polyfill,polyline);
    D(A--B--C--cycle,polyline);

    pair I=incenter(A,B,C);
    pair I_A=2*circumcenter(B,I,C)-I;
    path exc=CP(I_A,foot(I_A,B,C));
    path cir=circumcircle(A,B,C);
    pair F=IP(exc,cir,1);
    pair G=IP(exc,cir,0);
    D(exc); D(cir);
    pair D=foot(I_A,B,C),E=foot(A,B,C);
    pair M=(A+E)/2;
    D(B--foot(I_A,A,B)); D(C--foot(I_A,A,C));
    D(A--E);
    pair V=extension(D,G,M,M+B-E),N=extension(D,F,M,M+C-E);
    D(V--N); D(F--N); D(G--V); D(D--M);
    pair L=2*foot(I_A,M,D)-D;
    pair X=(D+L)/2;
    pair Y=extension(B,C,F,G);
    D(D--L); D(C--Y--L); D(Y--I_A); D(circumcircle(B,I,C));
    
    D("A",D(A),A);
    D("B",D(B),W);
    D("C",D(C),dir(0));
    D("I",D(I),W);
    D("I_A",D(I_A),S);
    D("F",D(F),dir(181));
    D("G",D(G),dir(-80));
    D("D",D(D),dir(-98));
    D("E",D(E),S);
    D("M",D(M),dir(48));
    D("V",D(V),W);
    D("N",D(N),dir(0));
    D("L",D(L),dir(0));
    D("X",D(X),dir(3));
    D("Y",D(Y),dir(45));
[/asy]

Let $V$ and $N$ be the intersections of $DG$ and $DF$ with the perpendicular bisector of $\overline{AE}$, let $L$ be the second intersection of line $MD$ with the $A$-excircle and let $M$ be the midpoint of $\overline{AE}$. It suffices to show $MV = MN$, or equivalently that $(VN; M\infty) = -1$. Note that it is well known that $M$, $I$ and $D$ are collinear.

Let $X$ be the midpoint of $\overline{DL}$ and $Y$ be $DD \cap LL$. Note that $\angle IXI_A = 90^\circ$ and thus $X$ lies on $(BIC)$. Additionally, $I_A L \perp YL$ and $XL \perp YI_A$, so we have \[YL^2 = YX \cdot YI_A = YB \cdot YC = YF \cdot YG\text{.}\]Thus $F$, $G$ and $Y$ are collinear and so $(DL; FG) = -1$. Projecting at $D$ gives us the desired result.
Z K Y
The post below has been deleted. Click to close.
This post has been deleted. Click here to see post.
dolphinday
1319 posts
#71 • 2 Y
Y by OronSH, cubres
Let $M$ be the midpoint of $AE$. Then our condition translates $M$ being the midpoint of $V$ and $N$, where $V$ and $N$ are the intersections of the perpendicular bisector of $AE$ with $DG$ and $DF$. Note that by EGMO $4.12$ we have $M$, $D$, and the incenter $I$ collinear. Let $Y$ be the second intersection of $MD$ with the excircle. Since we have $(V, N; M, P_{\infty}) \overset{D}= (G, F; Y, D)$. We wish to show that $GFYD$ is a harmonic quadrilateral, or that the tangents at $F$ and $G$ wrt the excircle concur with $YD$. Let circle $(II_A)$ intersect the excircle at $P$ and $Q$. Then by Fact $5$, $(II_A)$ passes through $B$ and $C$. Notice that the pole of $XY$ is $I$, which implies that the polar of $R$ passes through $I$(by La Hire's), which implies that $GYFD$ is harmonic, so we are done.
Z K Y
The post below has been deleted. Click to close.
This post has been deleted. Click here to see post.
jp62
53 posts
#72 • 2 Y
Y by OronSH, cubres
https://i.imgur.com/32VQdjt.jpeg
Diagram has points $V$ and $N$ omitted.
Setup
Finish

This was my last problem solved before 2024 IMO :) and a really fun one too. Haven't seen this trick in the finish before.
Z K Y
The post below has been deleted. Click to close.
This post has been deleted. Click here to see post.
EpicBird08
1745 posts
#73 • 1 Y
Y by cubres
oops

Let $\ell$ be the perpendicular bisector of $AE,$ and let $DG$ and $DF$ intersect $\ell$ at points $V$ and $N,$ respectively. If $M$ is the midpoint of $AE,$ we will also show that $M$ is the midpoint of $VN,$ which will evidently show that $EVAN$ is a rhombus.

Let $I$ be the incenter; it is well-known that $M,I,D$ are collinear. Let the line $\overline{MID}$ intersect the $A$-excircle $\omega$ again at point $P$.

Claim: (External version of ISL 2002 G7) $(BCP)$ is tangent to $\omega$.
Proof: Let $(BCP)$ intersect $ID$ again at point $Z.$ We first show that $Z$ is the midpoint of $ID.$ Indeed, if $W$ is the midpoint of $DP,$ then Power of a Point implies $$DB \cdot DC = DZ \cdot DP = DI \cdot DW$$since $BICWI_A$ is cyclic ($\angle IWI_A = 90^\circ$). Since $DP = 2DW,$ we see that $DI = 2DZ,$ implying that $Z$ is the midpoint of $DI.$ Then the Shooting Lemma implies the result.

Now, let the tangent at $P$ to $\omega$ intersect $BC$ again at point $X.$ Radical center on $(ABC), \omega, (BCP)$ implies that $F,G,X$ are collinear. Noting that $XD$ is tangent to $\omega$ as well, we discover that $DFPG$ is harmonic. Then $$(F,G;P,D) \stackrel{D}{=} (N,V;M,\infty) = -1,$$and we are done.
This post has been edited 1 time. Last edited by EpicBird08, Aug 16, 2024, 6:15 PM
Z K Y
The post below has been deleted. Click to close.
This post has been deleted. Click here to see post.
bebebe
992 posts
#74 • 1 Y
Y by cubres
Instead of proving existance, lets try to construct a rhombus. Specifically, let $M$ be midpoint of $AE,$ and perpendicular bisector of $AE$ intersect $DF$ and $DG$ at $N$ and $V.$ We would like to prove $M$ is midpoint of $VN.$


Let the intersection between $DM$ and the $A$ excircle be $P.$ We want to prove $$-1=(V, N; M, P_{\infty})\overset{D}{=}(G, F; P, D).$$Let tangent at $DD$ and $FG$ intersect at $X.$ We want to show $DP$ or $DI$ (points $P, D, I, M$ are collinear follows from homothethy at $D$) is the polar of $X.$


Since $X \in DD,$ the polar of $D$ contains $X.$ Let's now consider the polar of $I.$ The polar is the line between the intersection between the circle with diameter $II_A$ and the excircle; let these intersections be $Y,Z$. We notice that by the incenter-excenter lemma, the midpoint of $II_A$ lies on $(ABC)$, and also $BCII_A$ is cyclic. Thus, by Radical Axis Theorem, $BC, FG, YZ$ concur, at $X$! Thus, the polar of $I$ contains $X.$ Hence, the polar of $X$ is $DI$, and we are done.
Z K Y
The post below has been deleted. Click to close.
This post has been deleted. Click here to see post.
joshualiu315
2513 posts
#75 • 2 Y
Y by dolphinday, cubres
First of all, redefine $V,N$ as the intersections of the perpendicular bisector of $\overline{AE}$ with $\overline{DF}$, $\overline{DG}$. Denote the midpoint of $\overline{AE}$, incenter, and $A$-excenter as $M$, $I$, and $I_A$, respectively. Let $\Omega$, $\omega$, and $\omega_A$ be the circumcenter, $(II_A)$, and $A$-excircle, respectively. Finally, suppose that $\omega$ intersects $\omega_A$ at points $P$ and $Q$, that $\overline{MD}$ intersects $\omega_A$ at a second point $K$, and that the radical center of $\Omega$, $\omega$, and $\omega_A$ is $X$.

We wish to show that $M$ is the midpoint of $\overline{VN}$, which is equivalent to

\[-1 = (V,N;M,\infty) \overset{D}{=} (F,G;K,D).\]
In other words, we wish to show that $DFKG$ is harmonic. Hence, it suffices to show that $\overline{XK}$ is tangent to $\omega_A$.


Claim: $DPKQ$ is harmonic

Proof: Recall that $I$ lies on $\overline{MD}$. Then, note that $\angle IPI_A = \angle IQI_A = 90^\circ$, which means $IP$ and $IQ$ are tangent to $\omega_A$. Hence, we are done. $\square$


Since $DPKQ$ is harmonic, we know that $\overline{DD}$, $\overline{PQ}$, $\overline{KK}$ concur, and we also know that the concurrency point is $X$. Therefore, $\overline{XK}$ is tangent to $\omega_A$ as desired. $\square$
Z K Y
The post below has been deleted. Click to close.
This post has been deleted. Click here to see post.
cosdealfa
27 posts
#76 • 4 Y
Y by SomeonesPenguin, pb_ana, Kaus_sgr, cubres
Solved with SomeonesPenguin :D


Denote by $\omega_A$ the A-excircle, let $I$ be the incenter and let $U$ and $V$ be the intersection of circle $(BIC)$ with $\omega_A$. Let $S$ be the midpoint of $AE$, $FG\cap BC=\{J\}$ and let $T$ be on $\omega_A$ such that $JT$ it tangent to $\omega_A$.

Claim 1. $S$, $I$, $D$ and $T$ are collinear.

Proof: Note that from homothety $D$, $I$ and $S$ are collinear. Note that $IU$ and $IV$ are tangent to $\omega_A$ and $J$ is the radical center of circles $\omega_A$, $(ABC)$ and $(BIC)$, so it lies on $UV$.

Since $JT$ and $JD$ are tangent to $\omega_A$, it follows that $UDVT$ is harmonic, hence $I$ lies on $DT$. $\square$

Finally, the conclusion follows by projecting the harmonic quadrilateral $FDGT$ through $D$ on the line parallel to $BC$ passing through $S$.$\blacksquare$
Z K Y
The post below has been deleted. Click to close.
This post has been deleted. Click here to see post.
abeot
123 posts
#77 • 2 Y
Y by centslordm, cubres
Let $M$ be the midpoint of $AE$, let $I$ be the incenter of $\triangle ABC$, let $I_A$ be the $A$-excenter of $\triangle ABC$, and let $T$ be the intersection of $FG$ and $BC$.

It is well-known that $BICI_A$ is cyclic. Let $(I_A)$ denote the circumcircle of $\triangle DFG$ and $(II_A)$ denote the circumcircle of $BICI_A$.

Claim: $T$ is the radical center of $(ABC)$, $(I_A)$ and $(II_A)$.
Proof. Note that $BC$ is the radical axis of $(ABC)$ and $(II_A)$ while $FG$ is the radical axis of $(ABC)$ and $(I_A)$. $\square$

Let $P$ be the intersection of $TI_A$ with $(II_A)$, where $P \neq II_A$.

Claim: $P$ lies on $DI$.
Proof. From Power of a Point, \[ TP \cdot TI_A = TB \cdot TC = TF \cdot TG = TD^2 \]then it follows $\triangle TPD \sim \triangle TDI_A$, and so \[ \angle TPD = \angle TDI_A = 90^\circ \]however, we also have \[ \angle TPI = 180^\circ - \angle I_API = 180^\circ - 90^\circ = 90^\circ \]hence $\angle TPD = \angle TPI$ as needed. $\square$

Yet $I_AT \perp DI$ and $T$ lies on the tangent of $D$ to $(I_A)$; hence $T$ is the pole of $DI$ with respect to $(I_A)$. Then $FG$ passes through the pole of $DI$, so if $DI$ intersects $(I_A)$ again at $X$ with $D \neq D$, then $DFXG$ is harmonic. This implies that $DI$ is the $D$-symmedian of $\triangle DFG$.

Claim: Let $V$ and $N$ be the intersections of the line through $M$ parallel to $BC$ with $DF$ and $DG$ respectively. Then $VNFG$ is cyclic.
Proof. This follows by Reim's theorem where $(DFG)$ is tangent to $BC$ yet $VN \parallel BC$; explicitly, directing angles modulo $\pi$, \[ \measuredangle VFG = \measuredangle DFG = \measuredangle BDG = \measuredangle VNG \]as needed. $\square$

Thus $\triangle DFG \sim \triangle DNV$ and since $DI$ is the $D$-symmedian in $\triangle DFG$ then $DI$ is the $D$-median in $\triangle DNV$.
Yet it is well-known that $D$, $I$ and $M$ are collinear. It follows $MV = MN$. Yet $MA = ME$ by definition as well. Thus $EVAN$ is a rhombus. $\blacksquare$
Z K Y
The post below has been deleted. Click to close.
This post has been deleted. Click here to see post.
ihategeo_1969
195 posts
#78 • 1 Y
Y by cubres
We will define some new points.

$\bullet$ Let $P$ be midpoint of $\overline{AE}$.
$\bullet$ Let $I$ be incenter and $\omega_A$ be $A$-excircle.
$\bullet$ Let $X=\overline{ID} \cap \omega_A$.
$\bullet$ Let $T=\overline{BC} \cap \overline{FG}$.

Let $\ell$ be $\perp$ bisector of $\overline{AE}$. Now we just need to prove that if $V=\overline{DG} \cap \ell$ and $N=\overline{DF} \cap \ell$ then $(V,N;P,\infty)=-1$.

By Midpoint of altitude lemma, we have $P$, $I$, $D$ collinear and hence projecting thru $D$ we just need to prove that $(D,X;F,G)=-1$ or we just need to prove that $\overline{TX}$ is tangent to $\omega_A$.

By Extraversion of IMO Shortlist 2002/G7, $(BCX)$ is tangent to $\omega_A$. So see that $T$ is radical center of $(BCX)$, $(ABC)$, $\omega_A$ and hence it lies on $\overline{XX}$, as required.
Z K Y
N Quick Reply
G
H
=
a